Mertens conjecture: Difference between revisions

From Wikipedia, the free encyclopedia
Content deleted Content added
chg wording in lead again...
→‎Disproof of the conjecture: include 1.281 result
Line 14: Line 14:
== Disproof of the conjecture ==
== Disproof of the conjecture ==


[[Thomas Jan Stieltjes|Stieltjes]] claimed in 1885 to have proven a weaker result, namely that <math>{M(n)\over \sqrt{n}}</math> was [[Bounded function|bounded]], but did not publish a proof.
[[Thomas Jan Stieltjes|Stieltjes]] claimed in 1885 to have proven a weaker result, namely that <math>m(n)=M(n)/\sqrt{n}</math> was [[Bounded function|bounded]], but did not publish a proof.


In 1985, [[Andrew Odlyzko]] and [[Herman te Riele]] proved the Mertens conjecture false. It was later shown that the first [[counterexample]] appears below exp(3.21{{e|64}}) ([[János Pintz|Pintz]] 1987) but above 10<sup>14</sup> (Kotnik and Van de Lune 2004). The upper bound has since been lowered to exp(1.59{{e|40}}) (Kotnik and Te Riele 2006), but no counterexample is explicitly known. The boundedness claim made by Stieltjes, while remarked upon as "very unlikely" in the 1985 paper cited above, has not been disproven ({{as of|2009|lc=on}}). The [[law of the iterated logarithm]] states that if
In 1985, [[Andrew Odlyzko]] and [[Herman te Riele]] proved the Mertens conjecture false. It was later shown that the first [[counterexample]] appears below exp(3.21{{e|64}}) ([[János Pintz|Pintz]] 1987) but above 10<sup>14</sup> (Kotnik and Van de Lune 2004). The upper bound has since been lowered to exp(1.59{{e|40}}) (Kotnik and Te Riele 2006), but no counterexample is explicitly known. The boundedness claim made by Stieltjes, while remarked upon as "very unlikely" in the 1985 paper cited above, has not been disproven ({{as of|2009|lc=on}}). The [[law of the iterated logarithm]] states that if
&mu; is replaced by a random sequence of 1s and &minus;1s then the order of growth of the partial sum of the first ''n'' terms is (with probability 1) about (''n'' log log ''n'')<sup>1/2</sup>, which suggests that the order of growth of ''M''(''n'')/''n''<sup>1/2</sup> might be somewhere around (log log ''n'')<sup>1/2</sup>. The actual order of growth may be somewhat smaller; it was conjectured by Gonek in the early 1990's that the order of growth of <math>M(n)/\sqrt{n}</math> was <math>(\log{\log{\log{n}}})^{5/4}</math>, which was proved by Ng (2004), conditional on the Riemann hypothesis and assuming certain conjectures about the averaged behavior of zeros the Riemann zeta function.<ref>{{cite web|url=http://www.cs.uleth.ca/~nathanng/RESEARCH/mobius2b.pdf|title=The distribution of the summatory function of the MÄobius function}}</ref>
&mu; is replaced by a random sequence of 1s and &minus;1s then the order of growth of the partial sum of the first ''n'' terms is (with probability 1) about (''n'' log log ''n'')<sup>1/2</sup>, which suggests that the order of growth of ''m''(''n'') might be somewhere around (log log ''n'')<sup>1/2</sup>. The actual order of growth may be somewhat smaller; it was conjectured by Gonek in the early 1990's that the order of growth of ''m''(''n'') was <math>(\log{\log{\log{n}}})^{5/4}</math>, which was proved by Ng (2004), conditional on the Riemann hypothesis and assuming certain conjectures about the averaged behavior of zeros the Riemann zeta function.<ref>{{cite web|url=http://www.cs.uleth.ca/~nathanng/RESEARCH/mobius2b.pdf|title=The distribution of the summatory function of the MÄobius function}}</ref>


In 1979 Cohen and Dress found the largest known value of |''M''(''n'')|/''n''<sup>1/2</sup> =~ 0.570591 for <math>M(7766842813) = 50286</math>. In 2003 Kotnik and van de Lune extended the search to ''n'' = 10<sup>14</sup> but did not find larger values. In 2011 Kuznetsov claimed that |''M''(''n'')|/''n''<sup>1/2</sup> reached 0.585768 for <math>M(11,609,864,264,058,592,345)=-1,995,900,927</math>.<ref>{{cite web|url=http://arxiv.org/abs/1108.0135 |title="Computing the Mertens function on a GPU"| author =Eugene Kuznetsov|year =2011}}</ref>
In 1979 Cohen and Dress found the largest known value of ''m''(''n'') =~ 0.570591 for M(7766842813) = 50286. In 2003 Kotnik and van de Lune extended the search to ''n'' = 10<sup>14</sup> but did not find larger values. In 2006, Kotnik and te Riele improved the upper bound and showed that ''m''(233029271 5134531215 0140181996 7723401020 4456785091 6681557518 6743434036 9240230890 8933261706 9029233958 2730162362.807965)=1.218429.<ref>{{cite conference |url= http://www.springerlink.com/content/q0717243567v503t/ |title=The Mertens Conjecture Revisited |author=|authorlink= |coauthors= Tadej Kotnik and Herman te Riele|date= 2006|publisher= Springer |booktitle= Algorithmic Number Theory, 7th International Symposium |pages= 156-167|location=Berlin,Germany |id=10.1007/11792086_12 }}</ref>


== Connection to the Riemann hypothesis ==
== Connection to the Riemann hypothesis ==

Revision as of 12:08, 22 March 2012

In mathematics, the Mertens conjecture is the incorrect statement that the Mertens function M(n) is bounded by √n, which implies the Riemann hypothesis. It was conjectured by Stieltjes in a 1885 letter to Hermite (reprinted in Stieltjes 1905) and Mertens (1897), and disproved by Odlyzko & te Riele (1985). It is a striking example of a mathematical proof contradicting a large amount of computational evidence in favor of a conjecture.

Definition

In number theory, if we define the Mertens function as

where μ(k) is the Möbius function, then the Mertens conjecture is that for all n > 1,

Disproof of the conjecture

Stieltjes claimed in 1885 to have proven a weaker result, namely that was bounded, but did not publish a proof.

In 1985, Andrew Odlyzko and Herman te Riele proved the Mertens conjecture false. It was later shown that the first counterexample appears below exp(3.21×1064) (Pintz 1987) but above 1014 (Kotnik and Van de Lune 2004). The upper bound has since been lowered to exp(1.59×1040) (Kotnik and Te Riele 2006), but no counterexample is explicitly known. The boundedness claim made by Stieltjes, while remarked upon as "very unlikely" in the 1985 paper cited above, has not been disproven (as of 2009). The law of the iterated logarithm states that if μ is replaced by a random sequence of 1s and −1s then the order of growth of the partial sum of the first n terms is (with probability 1) about (n log log n)1/2, which suggests that the order of growth of m(n) might be somewhere around (log log n)1/2. The actual order of growth may be somewhat smaller; it was conjectured by Gonek in the early 1990's that the order of growth of m(n) was , which was proved by Ng (2004), conditional on the Riemann hypothesis and assuming certain conjectures about the averaged behavior of zeros the Riemann zeta function.[1]

In 1979 Cohen and Dress found the largest known value of m(n) =~ 0.570591 for M(7766842813) = 50286. In 2003 Kotnik and van de Lune extended the search to n = 1014 but did not find larger values. In 2006, Kotnik and te Riele improved the upper bound and showed that m(233029271 5134531215 0140181996 7723401020 4456785091 6681557518 6743434036 9240230890 8933261706 9029233958 2730162362.807965)=1.218429.[2]

Connection to the Riemann hypothesis

The connection to the Riemann hypothesis is based on the Dirichlet series for the reciprocal of the Riemann zeta function,

valid in the region . We can rewrite this as a Stieltjes integral

and after integrating by parts, obtain the reciprocal of the zeta function as a Mellin transform

Using the Mellin inversion theorem we now can express M in terms of 1/ζ as

which is valid for 1 < σ < 2, and valid for 1/2 < σ < 2 on the Riemann hypothesis. From this, the Mellin transform integral must be convergent, and hence M(x) must be O(xe) for every exponent e greater than 1/2. From this it follows that

for all positive ε is equivalent to the Riemann hypothesis, which therefore would have followed from the stronger Mertens hypothesis, and follows from the hypothesis of Stieltjes that

.

References

  1. ^ "The distribution of the summatory function of the MÄobius function" (PDF).
  2. ^ "The Mertens Conjecture Revisited". Algorithmic Number Theory, 7th International Symposium. Berlin,Germany: Springer. 2006. pp. 156–167. 10.1007/11792086_12. {{cite conference}}: Unknown parameter |booktitle= ignored (|book-title= suggested) (help); Unknown parameter |coauthors= ignored (|author= suggested) (help)